Which of the following statements is correct?:
A.). When expected rate of return is greater than required rate of return, this stock is overpriced.
B.). The Beta of treasury bill is 0.
C.). Systematic risk can be eliminated through diversification.
Option (B) is correct
The Beta of treasury bill is 0.
Beta measures the volatility of the stock returns with the market returns. Treasury bills are risk free assets / rates. Their rates do not change. So, there is less or no volatility of their returns.
Option (A) is incorrect because when the expected rate of return is greater than required rate of return, then the stock is underpriced.
Option (C) is incorrect because it is unsystematic risk, an not systematic risk that can be eliminated through diversification.
Get Answers For Free
Most questions answered within 1 hours.